There exists a permutation $(A_1, ldots , A_n)$ of the vertices $V ={1,ldots,n}$ such that for all...











up vote
1
down vote

favorite












Let $G = (V,E)$ be a directed graph on $n$ vertices, $V = {1, ldots,n}$, such that for every pair of distinct vertices $i, j in V$, exactly one of the two possible directed edges $(i, j)$ or $(j, i)$ is in the edge set $E$ (but not both). Prove that there must exist a permutation $(A_1, ldots, A_n)$ of the vertices $V ={1, ldots,n}$,such that for all $iin {1, ldots,n−1}, (A_i,A_{i+1})in E$.



I can think of how it is true but got stuck on proving it. Could anyone please help?










share|cite|improve this question









New contributor




yorkliang is a new contributor to this site. Take care in asking for clarification, commenting, and answering.
Check out our Code of Conduct.




















  • Where did you get stuck? If you show your thought process so far, we can better help you.
    – Santana Afton
    Nov 20 at 20:05










  • Start with a base case where you have 2 vertices, show that it works, use induction over the number of vertices.
    – B.Swan
    Nov 20 at 20:19








  • 1




    Furthermore, there is a unique permutation $(A_1,A_2,ldots,A_n)$ such that $(A_1,A_2,ldots,A_n)$ forms a directed path in $G$ iff $G$ has no directed cycles (i.e., a closed path $(v_1,v_2,ldots,v_l,v_1)$, where each $v_{i}v_{i+1}$ is an arc in $G$ for $i=1,2,ldots,l-1$, and $v_lv_1$ is also an arc in $G$). Equivalently, there exists a permutation $(A_1,A_2,dots,A_n)$ of vertices such that the in-degree of $A_i$ is $i-1$ for each $i=1,2,dots,n$. Equivalently, the relation $to$ on $V$ defined by $$uto v$$ iff there is a directed path in $G$ from $u$ to $v$ is an order relation on $V$.
    – Batominovski
    Nov 20 at 21:04

















up vote
1
down vote

favorite












Let $G = (V,E)$ be a directed graph on $n$ vertices, $V = {1, ldots,n}$, such that for every pair of distinct vertices $i, j in V$, exactly one of the two possible directed edges $(i, j)$ or $(j, i)$ is in the edge set $E$ (but not both). Prove that there must exist a permutation $(A_1, ldots, A_n)$ of the vertices $V ={1, ldots,n}$,such that for all $iin {1, ldots,n−1}, (A_i,A_{i+1})in E$.



I can think of how it is true but got stuck on proving it. Could anyone please help?










share|cite|improve this question









New contributor




yorkliang is a new contributor to this site. Take care in asking for clarification, commenting, and answering.
Check out our Code of Conduct.




















  • Where did you get stuck? If you show your thought process so far, we can better help you.
    – Santana Afton
    Nov 20 at 20:05










  • Start with a base case where you have 2 vertices, show that it works, use induction over the number of vertices.
    – B.Swan
    Nov 20 at 20:19








  • 1




    Furthermore, there is a unique permutation $(A_1,A_2,ldots,A_n)$ such that $(A_1,A_2,ldots,A_n)$ forms a directed path in $G$ iff $G$ has no directed cycles (i.e., a closed path $(v_1,v_2,ldots,v_l,v_1)$, where each $v_{i}v_{i+1}$ is an arc in $G$ for $i=1,2,ldots,l-1$, and $v_lv_1$ is also an arc in $G$). Equivalently, there exists a permutation $(A_1,A_2,dots,A_n)$ of vertices such that the in-degree of $A_i$ is $i-1$ for each $i=1,2,dots,n$. Equivalently, the relation $to$ on $V$ defined by $$uto v$$ iff there is a directed path in $G$ from $u$ to $v$ is an order relation on $V$.
    – Batominovski
    Nov 20 at 21:04















up vote
1
down vote

favorite









up vote
1
down vote

favorite











Let $G = (V,E)$ be a directed graph on $n$ vertices, $V = {1, ldots,n}$, such that for every pair of distinct vertices $i, j in V$, exactly one of the two possible directed edges $(i, j)$ or $(j, i)$ is in the edge set $E$ (but not both). Prove that there must exist a permutation $(A_1, ldots, A_n)$ of the vertices $V ={1, ldots,n}$,such that for all $iin {1, ldots,n−1}, (A_i,A_{i+1})in E$.



I can think of how it is true but got stuck on proving it. Could anyone please help?










share|cite|improve this question









New contributor




yorkliang is a new contributor to this site. Take care in asking for clarification, commenting, and answering.
Check out our Code of Conduct.











Let $G = (V,E)$ be a directed graph on $n$ vertices, $V = {1, ldots,n}$, such that for every pair of distinct vertices $i, j in V$, exactly one of the two possible directed edges $(i, j)$ or $(j, i)$ is in the edge set $E$ (but not both). Prove that there must exist a permutation $(A_1, ldots, A_n)$ of the vertices $V ={1, ldots,n}$,such that for all $iin {1, ldots,n−1}, (A_i,A_{i+1})in E$.



I can think of how it is true but got stuck on proving it. Could anyone please help?







combinatorics discrete-mathematics graph-theory directed-graphs






share|cite|improve this question









New contributor




yorkliang is a new contributor to this site. Take care in asking for clarification, commenting, and answering.
Check out our Code of Conduct.











share|cite|improve this question









New contributor




yorkliang is a new contributor to this site. Take care in asking for clarification, commenting, and answering.
Check out our Code of Conduct.









share|cite|improve this question




share|cite|improve this question








edited Nov 20 at 20:48









Batominovski

31.6k23188




31.6k23188






New contributor




yorkliang is a new contributor to this site. Take care in asking for clarification, commenting, and answering.
Check out our Code of Conduct.









asked Nov 20 at 19:58









yorkliang

61




61




New contributor




yorkliang is a new contributor to this site. Take care in asking for clarification, commenting, and answering.
Check out our Code of Conduct.





New contributor





yorkliang is a new contributor to this site. Take care in asking for clarification, commenting, and answering.
Check out our Code of Conduct.






yorkliang is a new contributor to this site. Take care in asking for clarification, commenting, and answering.
Check out our Code of Conduct.












  • Where did you get stuck? If you show your thought process so far, we can better help you.
    – Santana Afton
    Nov 20 at 20:05










  • Start with a base case where you have 2 vertices, show that it works, use induction over the number of vertices.
    – B.Swan
    Nov 20 at 20:19








  • 1




    Furthermore, there is a unique permutation $(A_1,A_2,ldots,A_n)$ such that $(A_1,A_2,ldots,A_n)$ forms a directed path in $G$ iff $G$ has no directed cycles (i.e., a closed path $(v_1,v_2,ldots,v_l,v_1)$, where each $v_{i}v_{i+1}$ is an arc in $G$ for $i=1,2,ldots,l-1$, and $v_lv_1$ is also an arc in $G$). Equivalently, there exists a permutation $(A_1,A_2,dots,A_n)$ of vertices such that the in-degree of $A_i$ is $i-1$ for each $i=1,2,dots,n$. Equivalently, the relation $to$ on $V$ defined by $$uto v$$ iff there is a directed path in $G$ from $u$ to $v$ is an order relation on $V$.
    – Batominovski
    Nov 20 at 21:04




















  • Where did you get stuck? If you show your thought process so far, we can better help you.
    – Santana Afton
    Nov 20 at 20:05










  • Start with a base case where you have 2 vertices, show that it works, use induction over the number of vertices.
    – B.Swan
    Nov 20 at 20:19








  • 1




    Furthermore, there is a unique permutation $(A_1,A_2,ldots,A_n)$ such that $(A_1,A_2,ldots,A_n)$ forms a directed path in $G$ iff $G$ has no directed cycles (i.e., a closed path $(v_1,v_2,ldots,v_l,v_1)$, where each $v_{i}v_{i+1}$ is an arc in $G$ for $i=1,2,ldots,l-1$, and $v_lv_1$ is also an arc in $G$). Equivalently, there exists a permutation $(A_1,A_2,dots,A_n)$ of vertices such that the in-degree of $A_i$ is $i-1$ for each $i=1,2,dots,n$. Equivalently, the relation $to$ on $V$ defined by $$uto v$$ iff there is a directed path in $G$ from $u$ to $v$ is an order relation on $V$.
    – Batominovski
    Nov 20 at 21:04


















Where did you get stuck? If you show your thought process so far, we can better help you.
– Santana Afton
Nov 20 at 20:05




Where did you get stuck? If you show your thought process so far, we can better help you.
– Santana Afton
Nov 20 at 20:05












Start with a base case where you have 2 vertices, show that it works, use induction over the number of vertices.
– B.Swan
Nov 20 at 20:19






Start with a base case where you have 2 vertices, show that it works, use induction over the number of vertices.
– B.Swan
Nov 20 at 20:19






1




1




Furthermore, there is a unique permutation $(A_1,A_2,ldots,A_n)$ such that $(A_1,A_2,ldots,A_n)$ forms a directed path in $G$ iff $G$ has no directed cycles (i.e., a closed path $(v_1,v_2,ldots,v_l,v_1)$, where each $v_{i}v_{i+1}$ is an arc in $G$ for $i=1,2,ldots,l-1$, and $v_lv_1$ is also an arc in $G$). Equivalently, there exists a permutation $(A_1,A_2,dots,A_n)$ of vertices such that the in-degree of $A_i$ is $i-1$ for each $i=1,2,dots,n$. Equivalently, the relation $to$ on $V$ defined by $$uto v$$ iff there is a directed path in $G$ from $u$ to $v$ is an order relation on $V$.
– Batominovski
Nov 20 at 21:04






Furthermore, there is a unique permutation $(A_1,A_2,ldots,A_n)$ such that $(A_1,A_2,ldots,A_n)$ forms a directed path in $G$ iff $G$ has no directed cycles (i.e., a closed path $(v_1,v_2,ldots,v_l,v_1)$, where each $v_{i}v_{i+1}$ is an arc in $G$ for $i=1,2,ldots,l-1$, and $v_lv_1$ is also an arc in $G$). Equivalently, there exists a permutation $(A_1,A_2,dots,A_n)$ of vertices such that the in-degree of $A_i$ is $i-1$ for each $i=1,2,dots,n$. Equivalently, the relation $to$ on $V$ defined by $$uto v$$ iff there is a directed path in $G$ from $u$ to $v$ is an order relation on $V$.
– Batominovski
Nov 20 at 21:04












1 Answer
1






active

oldest

votes

















up vote
1
down vote













We show it by induction on the number of vertices in the graph $G$.



Base case: when there is only two vertices, the claim is true.
Assume now that for graph with $n$ vertices, the claim is true, we have to show that a graph on $n+1$ vertices, the claim is true.



So let $G$ be a graph on $n+1$ vertices ${v_1,...,v_{n+1}$}. let's look at the vertex $v_{n+1}$,



Case 1:suppose it is adjacent only to outgoing edges. Then by induction hypothesis, the graph induced by vertices ${v_1,...,v_n}$ has a directed path, says, $v_1,...,v_n$. Well now, $v_{n+1},v_1,...,v_n$ is a directed path in $G$.



Case2: Suppose $v_{n+1}$ is adjacent to only ingoing edges, then by induction hypothesis, the graph induced by vertices ${v_1,...,v_n}$ has a directed path, says, $v_1,...,v_n$. Well now, $v_1,...,v_n,v_{n+1}$ is a directed path in $G$.



Case3: Suppose it is not case 1 nor case 2, then again by induction hypothesis, the graph induced by vertices ${v_1,...,v_n}$ has a directed path, says, $v_1,...,v_n$. Suppose we have edge $(v_1,v_{n+1})$ and for all $i$, we have edges $(v_iv_{n+1})$ and $(v_{i+1}v_{n+1})$ for $1leq i leq n-1$, we are back to case 2. So there must be $1 leq i leq n-1$ such that $(v_i,v_{n+1})$ and $(v_{n+1},v_{i+1})$ are edges in the graph. Well, $v_1,...,v_i,v_{n+1},v_{i+1},...,v_n$ is a hamiltonian directed path! done






share|cite|improve this answer























    Your Answer





    StackExchange.ifUsing("editor", function () {
    return StackExchange.using("mathjaxEditing", function () {
    StackExchange.MarkdownEditor.creationCallbacks.add(function (editor, postfix) {
    StackExchange.mathjaxEditing.prepareWmdForMathJax(editor, postfix, [["$", "$"], ["\\(","\\)"]]);
    });
    });
    }, "mathjax-editing");

    StackExchange.ready(function() {
    var channelOptions = {
    tags: "".split(" "),
    id: "69"
    };
    initTagRenderer("".split(" "), "".split(" "), channelOptions);

    StackExchange.using("externalEditor", function() {
    // Have to fire editor after snippets, if snippets enabled
    if (StackExchange.settings.snippets.snippetsEnabled) {
    StackExchange.using("snippets", function() {
    createEditor();
    });
    }
    else {
    createEditor();
    }
    });

    function createEditor() {
    StackExchange.prepareEditor({
    heartbeatType: 'answer',
    convertImagesToLinks: true,
    noModals: true,
    showLowRepImageUploadWarning: true,
    reputationToPostImages: 10,
    bindNavPrevention: true,
    postfix: "",
    imageUploader: {
    brandingHtml: "Powered by u003ca class="icon-imgur-white" href="https://imgur.com/"u003eu003c/au003e",
    contentPolicyHtml: "User contributions licensed under u003ca href="https://creativecommons.org/licenses/by-sa/3.0/"u003ecc by-sa 3.0 with attribution requiredu003c/au003e u003ca href="https://stackoverflow.com/legal/content-policy"u003e(content policy)u003c/au003e",
    allowUrls: true
    },
    noCode: true, onDemand: true,
    discardSelector: ".discard-answer"
    ,immediatelyShowMarkdownHelp:true
    });


    }
    });






    yorkliang is a new contributor. Be nice, and check out our Code of Conduct.










     

    draft saved


    draft discarded


















    StackExchange.ready(
    function () {
    StackExchange.openid.initPostLogin('.new-post-login', 'https%3a%2f%2fmath.stackexchange.com%2fquestions%2f3006810%2fthere-exists-a-permutation-a-1-ldots-a-n-of-the-vertices-v-1-ldots%23new-answer', 'question_page');
    }
    );

    Post as a guest















    Required, but never shown

























    1 Answer
    1






    active

    oldest

    votes








    1 Answer
    1






    active

    oldest

    votes









    active

    oldest

    votes






    active

    oldest

    votes








    up vote
    1
    down vote













    We show it by induction on the number of vertices in the graph $G$.



    Base case: when there is only two vertices, the claim is true.
    Assume now that for graph with $n$ vertices, the claim is true, we have to show that a graph on $n+1$ vertices, the claim is true.



    So let $G$ be a graph on $n+1$ vertices ${v_1,...,v_{n+1}$}. let's look at the vertex $v_{n+1}$,



    Case 1:suppose it is adjacent only to outgoing edges. Then by induction hypothesis, the graph induced by vertices ${v_1,...,v_n}$ has a directed path, says, $v_1,...,v_n$. Well now, $v_{n+1},v_1,...,v_n$ is a directed path in $G$.



    Case2: Suppose $v_{n+1}$ is adjacent to only ingoing edges, then by induction hypothesis, the graph induced by vertices ${v_1,...,v_n}$ has a directed path, says, $v_1,...,v_n$. Well now, $v_1,...,v_n,v_{n+1}$ is a directed path in $G$.



    Case3: Suppose it is not case 1 nor case 2, then again by induction hypothesis, the graph induced by vertices ${v_1,...,v_n}$ has a directed path, says, $v_1,...,v_n$. Suppose we have edge $(v_1,v_{n+1})$ and for all $i$, we have edges $(v_iv_{n+1})$ and $(v_{i+1}v_{n+1})$ for $1leq i leq n-1$, we are back to case 2. So there must be $1 leq i leq n-1$ such that $(v_i,v_{n+1})$ and $(v_{n+1},v_{i+1})$ are edges in the graph. Well, $v_1,...,v_i,v_{n+1},v_{i+1},...,v_n$ is a hamiltonian directed path! done






    share|cite|improve this answer



























      up vote
      1
      down vote













      We show it by induction on the number of vertices in the graph $G$.



      Base case: when there is only two vertices, the claim is true.
      Assume now that for graph with $n$ vertices, the claim is true, we have to show that a graph on $n+1$ vertices, the claim is true.



      So let $G$ be a graph on $n+1$ vertices ${v_1,...,v_{n+1}$}. let's look at the vertex $v_{n+1}$,



      Case 1:suppose it is adjacent only to outgoing edges. Then by induction hypothesis, the graph induced by vertices ${v_1,...,v_n}$ has a directed path, says, $v_1,...,v_n$. Well now, $v_{n+1},v_1,...,v_n$ is a directed path in $G$.



      Case2: Suppose $v_{n+1}$ is adjacent to only ingoing edges, then by induction hypothesis, the graph induced by vertices ${v_1,...,v_n}$ has a directed path, says, $v_1,...,v_n$. Well now, $v_1,...,v_n,v_{n+1}$ is a directed path in $G$.



      Case3: Suppose it is not case 1 nor case 2, then again by induction hypothesis, the graph induced by vertices ${v_1,...,v_n}$ has a directed path, says, $v_1,...,v_n$. Suppose we have edge $(v_1,v_{n+1})$ and for all $i$, we have edges $(v_iv_{n+1})$ and $(v_{i+1}v_{n+1})$ for $1leq i leq n-1$, we are back to case 2. So there must be $1 leq i leq n-1$ such that $(v_i,v_{n+1})$ and $(v_{n+1},v_{i+1})$ are edges in the graph. Well, $v_1,...,v_i,v_{n+1},v_{i+1},...,v_n$ is a hamiltonian directed path! done






      share|cite|improve this answer

























        up vote
        1
        down vote










        up vote
        1
        down vote









        We show it by induction on the number of vertices in the graph $G$.



        Base case: when there is only two vertices, the claim is true.
        Assume now that for graph with $n$ vertices, the claim is true, we have to show that a graph on $n+1$ vertices, the claim is true.



        So let $G$ be a graph on $n+1$ vertices ${v_1,...,v_{n+1}$}. let's look at the vertex $v_{n+1}$,



        Case 1:suppose it is adjacent only to outgoing edges. Then by induction hypothesis, the graph induced by vertices ${v_1,...,v_n}$ has a directed path, says, $v_1,...,v_n$. Well now, $v_{n+1},v_1,...,v_n$ is a directed path in $G$.



        Case2: Suppose $v_{n+1}$ is adjacent to only ingoing edges, then by induction hypothesis, the graph induced by vertices ${v_1,...,v_n}$ has a directed path, says, $v_1,...,v_n$. Well now, $v_1,...,v_n,v_{n+1}$ is a directed path in $G$.



        Case3: Suppose it is not case 1 nor case 2, then again by induction hypothesis, the graph induced by vertices ${v_1,...,v_n}$ has a directed path, says, $v_1,...,v_n$. Suppose we have edge $(v_1,v_{n+1})$ and for all $i$, we have edges $(v_iv_{n+1})$ and $(v_{i+1}v_{n+1})$ for $1leq i leq n-1$, we are back to case 2. So there must be $1 leq i leq n-1$ such that $(v_i,v_{n+1})$ and $(v_{n+1},v_{i+1})$ are edges in the graph. Well, $v_1,...,v_i,v_{n+1},v_{i+1},...,v_n$ is a hamiltonian directed path! done






        share|cite|improve this answer














        We show it by induction on the number of vertices in the graph $G$.



        Base case: when there is only two vertices, the claim is true.
        Assume now that for graph with $n$ vertices, the claim is true, we have to show that a graph on $n+1$ vertices, the claim is true.



        So let $G$ be a graph on $n+1$ vertices ${v_1,...,v_{n+1}$}. let's look at the vertex $v_{n+1}$,



        Case 1:suppose it is adjacent only to outgoing edges. Then by induction hypothesis, the graph induced by vertices ${v_1,...,v_n}$ has a directed path, says, $v_1,...,v_n$. Well now, $v_{n+1},v_1,...,v_n$ is a directed path in $G$.



        Case2: Suppose $v_{n+1}$ is adjacent to only ingoing edges, then by induction hypothesis, the graph induced by vertices ${v_1,...,v_n}$ has a directed path, says, $v_1,...,v_n$. Well now, $v_1,...,v_n,v_{n+1}$ is a directed path in $G$.



        Case3: Suppose it is not case 1 nor case 2, then again by induction hypothesis, the graph induced by vertices ${v_1,...,v_n}$ has a directed path, says, $v_1,...,v_n$. Suppose we have edge $(v_1,v_{n+1})$ and for all $i$, we have edges $(v_iv_{n+1})$ and $(v_{i+1}v_{n+1})$ for $1leq i leq n-1$, we are back to case 2. So there must be $1 leq i leq n-1$ such that $(v_i,v_{n+1})$ and $(v_{n+1},v_{i+1})$ are edges in the graph. Well, $v_1,...,v_i,v_{n+1},v_{i+1},...,v_n$ is a hamiltonian directed path! done







        share|cite|improve this answer














        share|cite|improve this answer



        share|cite|improve this answer








        edited Nov 20 at 20:53









        Batominovski

        31.6k23188




        31.6k23188










        answered Nov 20 at 20:21









        mathnoob

        92813




        92813






















            yorkliang is a new contributor. Be nice, and check out our Code of Conduct.










             

            draft saved


            draft discarded


















            yorkliang is a new contributor. Be nice, and check out our Code of Conduct.













            yorkliang is a new contributor. Be nice, and check out our Code of Conduct.












            yorkliang is a new contributor. Be nice, and check out our Code of Conduct.















             


            draft saved


            draft discarded














            StackExchange.ready(
            function () {
            StackExchange.openid.initPostLogin('.new-post-login', 'https%3a%2f%2fmath.stackexchange.com%2fquestions%2f3006810%2fthere-exists-a-permutation-a-1-ldots-a-n-of-the-vertices-v-1-ldots%23new-answer', 'question_page');
            }
            );

            Post as a guest















            Required, but never shown





















































            Required, but never shown














            Required, but never shown












            Required, but never shown







            Required, but never shown

































            Required, but never shown














            Required, but never shown












            Required, but never shown







            Required, but never shown







            Popular posts from this blog

            Wiesbaden

            To store a contact into the json file from server.js file using a class in NodeJS

            Marschland